LSAT and Law School Admissions Forum

Get expert LSAT preparation and law school admissions advice from PowerScore Test Preparation.

 Administrator
PowerScore Staff
  • PowerScore Staff
  • Posts: 8929
  • Joined: Feb 02, 2011
|
#91288
Complete Question Explanation

The correct answer choice is (D).

Answer choice (A):

Answer choice (B):

Answer choice (C):

Answer choice (D): This is the correct answer choice.

Answer choice (E):

This explanation is still in progress. Please post any questions below!
User avatar
 rdee81
  • Posts: 18
  • Joined: Aug 23, 2021
|
#95048
I have to go through all the answer choices to find the right answer by plugging letters in. This time consuming. What is a better way?
 Rachael Wilkenfeld
PowerScore Staff
  • PowerScore Staff
  • Posts: 1392
  • Joined: Dec 15, 2011
|
#95081
Hi rdee,

You need to make a local diagram here, and see what the implications are.

So if F is before J, F must be before MJ. Connecting that local rule with our main diagram, we get

L :longline: K :longline: F :longline: MJ.

At first, it looks like H could go anywhere other than between M and J. However, there are a few slots we can rule out. First, we can rule out H in 1, because that would require L in 4. That wouldn't leave room for L-K-F-MJ after 4. Unfortunately, first is not an answer choice here.

So where else can't H go? We know it can't go between M and J, but we don't know where the MJ block goes yet. Let's look at the options. The earliest MJ can go would be fourth and fifth (putting LKF all before MJ). The latest MJ can go would be fifth and sixth, putting LKF and H somewhere before the MJ block. But look! In both situations, MJ takes up slot 5! That means no matter what, slot 5 is full, and cannot be H. I'd look for fifth as an answer choice, select it, and move on.

Hope that helps!

Get the most out of your LSAT Prep Plus subscription.

Analyze and track your performance with our Testing and Analytics Package.